« first day (31 days earlier)      last day (543 days later) » 
00:00 - 14:0014:00 - 00:00

2:25 PM
0
Q: $n points on circle how many different square and Curved square$

Itai Taln points on circle how many different square and Curved square you can build My solution is: $C(n,4)$ to the curved squre and $C(n,4)*3$ to the squre which makes total sense, for me. and I know it's not enough

A title should not be all-MathJax; having some plain text helps with search and navigation. (from a bot)Normal Human 20 secs ago
0
Q: Real value of x in Image attached

Elantaenter image description here I have solved the above using the below method. x= 12 + 1/(2+(1/2+x)) After solving for x, i got the answer as 11.7515 and -1.41824 So what is the real value of x, it should be one value for the expression. Why am I getting two values? Please assist

Question contains please. Real value of x in Image attached
 
2
Q: Question closed as duplicate of question closed as too localized

Tim BI just saw this question: How to compare three doubles and find the maximum, minimum, and the medium? (In the shortest way possible) Closed as a duplicate of this question: The fastest way to sort 3 values Java That makes sense, the duplicate is clear. However the second question has itself be...

 
0
Q: Finding unitary matrix

ELECFind unitary matrix $U\in\mathbb{C}^{2\times2}$ so that $D=UAU^*$ is diagonal where $$A=\begin{bmatrix} 3 & -4\\4 &3 \end{bmatrix} \in \mathbb{R}^{2\times 2}$$ I know that for unitary matrix it holds that $U^* U = U U^* = I$ and if we know that $D$ is diagonal: \begin{align} D&= U^* A U \;\;\;|...

0
Q: Terence Tao Exercise 5.4.8: Boundedness of Limit problem.

user3135030Let $ \{ a_{n} \}_{n=1}^{\infty} $ be a Cauchy sequence of rationals, and let $x$ be a real number. Show that if $a_{n} \leq x $ for all $n \geq 1 $, then $\lim_{n \rightarrow \infty} a_{n} \leq x $.

0
Q: Is the following theorem true?

Birdman2246So, I am working on some analysis homework and created a lemma to help me prove something. The problem is I don't know if it's true or false, and I don't want to waste a whole bunch of time attempting to prove a false lemma. Is the following lemma true? Proofs are not required, but greatly apprec...

Consider replacing (analysis) with a more specific tag for the relevant branch of analysis. (from a bot)Normal Human 20 secs ago
 
2:50 PM
0
Q: Fractal is an uncountable set

shtlmdI read on internet that " Fractal is special example of Cantor set". My question, Is Cantor set uncountable? Hence, Fractal is also an example of uncountable set? Thanks

Consider replacing (analysis) with a more specific tag for the relevant branch of analysis. (autocomment)Normal Human 20 secs ago
0
Q: Angles in a triagle ABC

Pls2In a triangle ABC from vertex C median of a line segment AB, angle $\angle|BCA|$ bisector and line segment perpendicular to a line segment AB divides angle $\angle|BCA|$ to four equal parts. Task is to compute angles in a triangle ABC. Thanks for any help.

 
3:13 PM
0
Q: Proving a degree sequence satisfy Chvatal’s criterion

SmithHow can I prove that a degree sequence satisfy Chvatal’s criterion? I know that i must prove that sequence A is Hamiltonian if and only if A' is hamiltonian but i am lost on where i should start. for example does this satisfy Chvatal’s criterion? (3, 3, 4, 4, 4, 6, 7, 7, 8, 8)

Please don't use (self-learning) tag just because you were self-studying. This tag is only for questions about the process of self-studying. (from a bot)Normal Human 21 secs ago
0
Q: characteristic function proof help

Salim Shaikhhow to Prove that e^-i|x| is not a characteristic function

Welcome to Math.SE, Salim Shaikh. Words such as help do not add information to titles. Please edit the title so that it better describes the specifics of your question. Do not hesitate to make it longer or include a formula if needed. More tips here. (from a bot)Normal Human 21 secs ago
0
Q: Open Newton Cotes Problem

Morgan WeissConsider numerically approximating the integral $$I = \int_{a}^{b}f(x)dx$$ using the open Newton-Cotes with $n = 2$,i.e., 3 points $$I_{2}^{(o)} = \frac{4}{3}h_2[2f(x_0) - f(x_1) + 2f(x_2)]$$ 4b.) Suppose global step halving is used to define a coarse grid with $m$ intervals of size $H_c$ and a...

Short title. Title contains problem. Open Newton Cotes Problem
0
Q: Integrals and Related Rates

Basel Anani2 (2 pts) A window in Prof. Xu' ss office is by the function f where f is given by f (s) = 5 sin ((π/32)(s^2 + 16)). Every evening, before she leaves her office, Prof. Xu draws the curtain over the window. Every morning she draws the curtain open, starting at time t = 0. If she draws the cu...

0
Q: Real roots of a polynomial function

Billy McGeenI need to prove the for every 0 < k <3 there are at least 2 real roots to the fucntion: $f\left(x\right)=x^5-8x^2+k$

0
Q: What is the derivative of matrix vector product (Ax) with respect to A?

user3727929What is the derivative of a vector with respect to a matrix? Specifically, $\frac{d(Ax)}{dA} = ? $, where $ A \in R^{m \times n}$ and $x \in R^n$.

0
Q: is every irrational numbr a root of polynomial of degree 2

M.GareebHow to prove that every irrational number with eventually periodic continued fraction expansion is a root of a polynomial of degree 2?

Title ends with a digit. Short question. is every irrational numbr a root of polynomial of degree 2
0
Q: How to find the values of Q1 and Q2 Image attached

Gabriella Please let me know the values of Q1 and Q2 and what does this say? (Image attached)

Short question. Question contains please. How to find the values of Q1 and Q2 Image attached
0
Q: On $1+e^{x}+e^{2x}+e^{3x}+e^{4x}+e^{5x}$

Tito Piezas IIIDefine the following, $$F_2(x) := \frac{1}{2}+\frac{(2x)}{1!} B_2\Big(\tfrac{1}{2}\Big)+\frac{(2x)^2}{2!}B_3\Big(\tfrac{1}{2}\Big)+\frac{(2x)^3}{3!}B_4\Big(\tfrac{1}{2}\Big)+\dots \infty$$ $$\color{brown}{F_3(x)} := \frac{1}{3}+\frac{(3x)}{1!} B_2\Big(\tfrac{1}{3}\Big)+\frac{(3x)^2}{2!}B_3\Big(...

 
3:31 PM
0
Q: Computer-algebra tag?

benblumsmith Are questions about implementation of specific tasks in specific computer algebra systems on-topic here? If so, can we create a computer-algebra tag? Context: I have been feeling around on the StackExchange network for an appropriate place to ask nuts-and-bolts questions about how to im...

 
0
Q: Need help interpreting this symbol

tatanI came across this symbol while reading about number theory what does it mean? Thanks for any help.

0
Q: Prove that $\sum_{k=m}^n x_k>1-\dfrac{a_m}{a_n}\forall n\geq m $

AmartyaLet $ {a_k}$ be an unbounded,strictly increasing sequence of positive real numbers and $x_k=\dfrac{a_{k+1}-a_k}{a_{k+1}}$. Prove that $\sum_{k=m}^n x_k>1-\dfrac{a_m}{a_n}\forall n\geq m $ and $\sum_{k=1}^\infty x_k$ diverges to $+\infty.$ What I thought: $\sum_{k=m}^n x_k=\sum_{k=m}^n(1-\dfrac...

Words such as help are uninformative in titles. Please edit the title so that it better describes the specifics of your question. Do not hesitate to make it longer or include a formula if needed. More tips here. (from a bot)Normal Human 21 secs ago
0
Q: Find the supremum of

user135520I was looking to find the supremum of this set of real numbers $$\Big\{ \vert \sum^{\infty}_{n=1} \frac{a}{n^{2}+a^{2}} \vert: a \in \mathbb{R} \Big\}$$ I was able to show (I hope this is right) that for any $a \in \mathbb{R}$, that $\vert \sum^{\infty}_{n=1} \frac{a}{n^{2}+a^{2}} \vert< \vert...

Short title. Find the supremum of
0
Q: First order language for affine spaces theory

ValerioIs there a language of affine spaces theory ? Is it first order ?

0
Q: Order of $a^{220}?$

Rishabh SareenIf in a group $G$ there is an element $a$ of order $360$, what is the order of $a^{220}$ ? How to proceed ?

Short title. Short question. Order of $a^{220}?$
0
Q: Convexity of specific function - optimization problem

Marcos LeeI have the following simple optimization problem $$ \text{max} \ x^2 + 3xy + y^2 \text{ subject to } x + y = 100$$ and the exercise tells me to check whether I have found the actual solution by checking its convexity. The way I would solve would be to break the function into $(x+y)^2 + xy$ and...

0
Q: Help in changing parametric form to Cartesian form

xXLemonProgrammerXxI need to eliminate $t$ from, $$ x = t + \cos \frac{t}{k} $$ and $$y = -\sin \frac{t}{k}$$ to get an equation in terms of $x$ and $y$ only. Essentially, I am trying to convert it from parametric form to Cartesian form. Please help, I am stuck. :(

Words such as help are uninformative in titles. Please edit the title so that it better describes the specifics of your question. Do not hesitate to make it longer or include a formula if needed. More tips here. (from a bot)Normal Human 21 secs ago
0
Q: Proof of ABC conjecture

AlisaCan someone please help me prove this relation in ABC conjecture The ABC conjecture says that for every real number e > 0, there exist only finitely many triples (a; b; c) of positive integers, with gcd(a; b; c) = 1, that satisfy a + b = c and c > (rad(abc))^(1+e)

Consider adding a tag for a broader subject area to which the question belongs. Some of these tags might fit. (autocomment)Normal Human 21 secs ago
 
3:58 PM
0
Q: Merge/alias [stub] and [stubs]

krygerThere seems to be no reason to keep both of them separate: http://stackoverflow.com/questions/tagged/stub vs http://stackoverflow.com/questions/tagged/stubs

 
0
Q: Show that every finite subset of {0,1}* is recursive.

AltaïrHow can I show that every finite subset of $\{0,1\}^*$ is recursive ?

0
Q: cross product of E and H

user65652Can anyone explain how they came up with the product of E and H ? I don't understand why the exponent of E cross H are multiplied by 2. Thanks enter image description here

Questions tend to get more attention when they have a tag for a broad area of mathematics relevant to the question. Some of these tags might fit. (from a bot)Normal Human 21 secs ago
0
Q: Which spelling is correct or most common Lagrangian or Lagrangean?

abdeaitaliI often encounter both spellings Lagrangian or Lagrangean. I just want to know which one is correct or commonly used in mathematical journals.

0
Q: Problem related to cyclic and abelian group

user295574Let G be a group of order 3n for some $n\in\mathbb Z$. Suppose all the elements of G of order 3 are conjugate .Then which are true? G must be cyclic. G canot be abelian G must be abelian and not cyclic G must be abelian and may or may not be cyclic. Since all the elements of order 3 are conj...

0
Q: How to evaluate $\int_c \dfrac{dx+dy}{|x|+|y|}$ , where $C$ is a certain square traversed once in a counter-clockwise direction ?

Saun DevHow to evaluate $\int_c \dfrac{dx+dy}{|x|+|y|}$ , where $C$ is the square with vertices $(1,0);(0,1);(-1,0);(0,-1)$ traversed once in a counter-clockwise direction ?

Tall formulas in titles break the layout of question lists. Please replace \dfrac with \frac in the title. (autocomment)Normal Human 21 secs ago
0
Q: Determine $Q^+$:($Q^+$ is more penrose inverse of $Q$)

ghazaleLet each edge of $K_n$ be given an orientation and let $Q$ be the incidence matrix. Determine $Q^+$:($Q^+$ is more penrose inverse of $Q$)

0
Q: Partial fraction decomposition in $\mathbb C (X)$ of real fraction

palioLet $F={P\over Q} $ be a fraction with $deg(P)< deg(Q)$ and such that $P$ and $Q$ are polynomials with real coefficients. Suppose $Q$ has the form $Q=(X-a)(X-z)(X-\bar z)$ with $a\in \mathbb R$ and $z$ is a non real root of $Q$ and $\bar z$ is the conjugate of $z$. The partial fraction decomposi...

Consider adding a tag for a broader subject area to which the question belongs. Some of these tags might fit. (autocomment)Normal Human 21 secs ago
0
Q: Prove a∙〖10〗^n≡a∙1(mod 3) using induction.

rdeshaisI need to prove this equation to be true not sure how to solve. I know I have to first use one and then plug in k+1 but what am I plugging it into a or n and then how do I solve? Thanks

0
Q: Double complex integral

mcrouchSo basically I want to integrate over two complex variables, so my integration will look something like this $\int uv\cdot e^{-uv}dudv$ where u and v are complex coordinates, in this case two dimensional (u=x+iy) and v=(x-iy). Im wondering how to solve this integral. I have seen a number of dif...

0
Q: Fermat's theorem existance

Roberta BlaizeProve that Fermat's Last Theorem is true for all large n, i.e., for all large integers n>=3, the equation x^n + y^n = z^n has no solution in positive integers x, y and z.

Short title. Short question. Fermat's theorem existance
 
4:21 PM
0
Q: How might I better prepare myself for the next moderator election?

MosheI've been a Stack Overflow user for more than 6 years and I've seen many moderator elections. Like many, each election wakes up an urge inside to run for moderator. I'll usually write my pitch, using what other moderators have called out as valuable skills in addition to a few of my own. I've go...

 
0
Q: showing something is larger than n/2

Syed NaqiI need help with this prove it is part of my homework for school so I was stuck on this question and don't know how to do this so here is the question: enter image description here

Title ends with a digit. Short question. showing something is larger than n/2
0
Q: Absolute continuity of function.

student93I have to give an example of absolutely continuous function. Can somebody help? I know that if $f$ is constans on an interval, then $f$ is absolutely continuous.

0
Q: Is there a formula for the largest number less than $10^n$ that has 8 divisors?

user19405892Find a general formula for the largest number less than $10^n$ that has exactly $8$ divisors. I was wondering if there was a way to find a general formula for this or if it is possible to create one.

Consider adding a tag for a broader subject area to which the question belongs. Some of these tags might fit. (autocomment)Normal Human 21 secs ago
0
Q: How could I modify a Turing machine that crashes on some inputs so that it doesn't crash at all?

Out Of BoundSuppose that M is a TM that crashes on some inputs. How would I modify M so that the new machine accepts the same language but does not crash on any input ?

0
Q: Basic probabilities and statistics

dmgI received some question for my exam preparation, but I'm not sure about the answer. So I need some reasonable explanation of the following tasks: I. Every day Sonja arrives at the railway station between 6 and 7 pm, but her exact time of arrival is random within this one hour interval. The trai...

0
Q: Limits with sqrt 1

Olya12Limits: $$\lim\limits_{x\to1}{\frac{\sqrt{2^x+7}-\sqrt{2^{x+1}+5}}{x^3-1}}$$ I multiplied by the conjugate but nothing happened

Short title. Title ends with a digit. Short question. Limits with sqrt 1
0
Q: Openness of the inverse image of a real interval under a metric map

AjmalWLet $(X,d)$ me a metric space and pick a real interval $(a,b)$. Why is $d^{-1}((a,b))$ open in the product topology on $X\times X$ induced by the metric topology on $X$?

0
Q: Prove that $\lim_{x\to0^+}[1+[x]]^{\frac{2}{x}}=1$,where $[x]$ represents the floor function of $x$

Vinod Kumar PuniaProve that $\lim_{x\to0^+}[1+[x]]^{\frac{2}{x}}=1$,where $[x]$ represents the floor function of $x$ $\lim_{x\to0^+}[1+[x]]^{\frac{2}{x}}=\lim_{x\to0^+}[1]^{\frac{2}{x}}$ Because $\lim_{x\to0^+}[x]=0$ But i am stuck.Please help me.Thanks.

Question contains please. [Prove that $\lim_{x\to0^+}[1+[x]]^{\frac{2}{x}}=1$,where $[x]$ represents the floor function of $x$](math.stackexchange.com/q/1558364)
0
Q: Probability, Central Limit Theorem

StrikerThe probability that a seed will germinate is 0.47. Suppose 156 seeds are planted. Use the Central Limit Theorem to determine the probability that at most 72 seeds germinate

0
Q: Constructing a sample by correlation

RodelSuppose we have two samples with known correlation (should be relatively high). Say both samples have $n$ data points. What if now we still know the correlation factor but one sample only consistent of the first 5 data point. Could one still construct the remaining data points solely using the ...

0
Q: Without using L'hopital rule $\lim_{x \to \pi/2 }\frac{\cos3x}{1+\cos2x}$.

Lillycan you help me please with limit solving without using L'hopital rule $\lim_{x \to \pi/2 }\frac{\cos3x}{1+\cos2x}$. Thanks a lot!

Consider adding a tag for a broader subject area to which the question belongs. Some of these tags might fit. (autocomment)Normal Human 28 secs ago
0
Q: What kind of functions can we really build when we are dealing with the limit of piecewise linear continuous functions?

Ante PaladinOh, well, the title actually describes what kind of question will this question be, but let us do some warm-up before stating the question as clearly as possible. Suppose first that everything we do we do on some set of the form $[a,b]$. Suppose secondly that we have some sequence $f_k$ ; $k \i...

Consider replacing (analysis) with a more specific tag for the relevant branch of analysis. (autocomment)Normal Human 20 secs ago
 
Anonymous
4:46 PM
what is this room for?
 
0
Q: omega-limit set (Heteroclinic orbit)

S. MoroFor general nonlinear system, can a union of equilibrium point (sink) and Heteroclinic orbit be a omega-limit set. Thanks

0
Q: Maze algorithm question

Bob de BouwerI posted quite an interesting question yesterday about a maze algorithm, but nobody responded yet. Would you mind taking a look? Runtime of maze algorithm

Words such as question do not add information to titles. Please edit the title so that it better describes the specifics of your question. Do not hesitate to make it longer or include a formula if needed. More tips here. (autocomment)Normal Human 21 secs ago
 
5:06 PM
0
Q: Error in link inside tooltip of newest badge

JamesRecently, I earned the Enlightened badge. When I visit my own profile and click on the Newest badge, I see a tooltip with the text "Awarded for: How do I calculate someone's age in C#?", like the image below. The problem is that this is a mistake. When I click in "Learn More", I see another ...

 
0
Q: Question on induction regarding monochromatic triangles in graph colourings- Homework related

mathcompLet $k$ be a natural number and for each $k$ let $r_k$ be the minimum number $n$ so that if we colour the edges of $K_n$ with $k$ colours then we can find a monochromatic triangle. I have so far showed that $r_k − 1 ≤ k(r_{k-1} − 1) + 1$ and now I have been asked to do the following: Use induc...

0
Q: Moving around summations

user3701257Given an $n\times n$ non-negative with dominant eigenvalue $\mu$. Let $m>\mu$. Consider the infinite summation $$\sum_{k=0}^\infty \frac{M^k}{m^k}$$ Is the following equality correct: $$\min_{i\le j\le n} \sum_{i=1}^n\sum_{k=0}^\infty\frac{M^k}{m^k} = \sum_{k=0}^\infty \frac{1}{m^k} \min_{1\le...

0
Q: images of group homomorhpisms

saraI want to know the possible images of the group homomorphism that maps D10 --> G (where G is some arbitrary group) Things I know: G/ker(f) is iso to Im(f) normal subgroup to D10 are {e}, $<sigma>$, D10 I do not know how to connect all this information, so please help!

Short title. Question contains please. images of group homomorhpisms
 
5:24 PM
0
Q: Finding two non congruent right angle triangles

GabrielIn the image attached, enter image description here Can you help find two non-congruent right angled triangle that have rational side lengths and area equal to 5

 
0
Q: Does Keen IO Link to Stack Overflow Appropriately?

BlackVegetableI read the question regarding another company's use of a Stack Overflow link with the heading of "Community" and noticed that some of the way that company was referencing Stack Overflow was similar to what Keen IO (A company I currently work for) is doing. There are some differences in the way w...

 
0
Q: basic question product of projective space

userOn $\mathbb P^n_k\times_k \mathbb P^m_k$, is it true that $T_{\mathbb P^n_k\times_k \mathbb P^m_k}\otimes \mathcal O_{\mathbb P^n_k\times_k \mathbb P^m_k}(d,e)\simeq p_1^*T_{\mathbb P^n_k}(d)\oplus p_2^*T_{\mathbb P^m_k}(e)$ ? ($n,m\geq 1$ and $p_i$ are the two projection).

Words such as question do not add information to titles. Please edit the title so that it better describes the specifics of your question. Do not hesitate to make it longer or include a formula if needed. More tips here. (from a bot)Normal Human 20 secs ago
0
Q: Calc I Substitution/Integration - Prof. Couldn't Complete Problem

Gabrielle FI'm a first-time Calc I student currently struggling in class. Yesterday we started on Substitution and Integration with integrals. One problem our professor put on the board was: $\int \frac{\left(x^2+2.1x\right)}{\left(x^3+3x+12\right)^6}dx$ And he refused to solve it, said nobody in the room...

Questions tend to get more attention when they have a tag for a broad area of mathematics relevant to the question. Some of these tags might fit. (from a bot)Normal Human 21 secs ago
 
0
Q: expression to prove that the earth orbits the sun

Ihazah Hopea. First using Newton's laws; derive an expression that proves that Earth could not be in the center of the solar system. Keep your derivation detailed, neat, clean. All Algebra work must be consistent. (what i am thinking is if the question is asking to find how to get F=ma and also Newton's law...

0
Q: What strategy might I use to find a question to place a bounty on?

Tim BSo. I've been looking at the altruist and investor badges, and was wanting to add them to my collection. However I don't want to just put a bounty on any old rubbish question. Equally I don't want to just add yet more rep on an already highly up voted answer. So is there any strategy I can use t...

 
5:48 PM
0
Q: General Case set question

SamIf you have 2 sets, one being called A and another B, which B is a subset of A. You need to determine the formula in general for A to intersect B who contains only one element. My idea, was to take a and say that the cardinality of A = i, and the cardinality of B = j. Then devising a general ...

0
Q: Finding the tangent equation

gbox Find the tangent equation hows slop is $1$ and is tangent to $f(x)=18x^5-17$ I have got two tangent function: $y_a=x-17.25$ and $y_b=x-16.73$ can it be?

Short title. Short question. Finding the tangent equation
Welcome to Math.SE, Sam. Words such as question are uninformative in titles. Please edit the title so that it better describes the specifics of your question. Do not hesitate to make it longer or include a formula if needed. More tips here. (from a bot)Normal Human 21 secs ago
0
Q: Limit point of a sequence

user8Given a sequence $(a_n)_{n \in \mathbb{N}} \subset \mathbb{R}$ and a set $M = \{a_n : n \in \mathbb{N}\}$, is $|M| < \infty$, then $(a_n)_{n \in \mathbb{N}} $ has a limit point. I believe that $(a_n)_{n \in \mathbb{N}}$ possesses no limit points. Here is why: From the definition of a limit poin...

0
Q: Applied matrix problem with probabilities

ELECFor city we have simplified its weather forecasting as such. If it rains then the probability for rain the next day is $0.2$. If its sunny then the probability for sunny day the next day is $0.7$. Vector $$x_{k}=\begin{bmatrix}\text{probability for sunny weather at day } k \\ \text{probability f...

0
Q: Wolframalpha for Fourier series

MartinPlease is there a way to input this fourier series into wolfram alpha ? I was given: f(x)= x for (-pi, pi). So I need to calculate A0, An, Bn coefficients. Thanks for help. Martin

Question contains please. Wolframalpha for Fourier series
0
Q: method in finding absolute convergence of a given series

marg_ocruzHow do I show absolute convergence for the series $$\sum_{n=0}^{\infty} \frac{n}{\sqrt{2n^5 +1}}$$ I have already showed by Comparison test that it is convergent. I am after the way of showing $\sum |a_n|$ is convergent. I tried ratio and root test but it gives me a limit of 1 so I need to do a...

0
Q: Probability theory question about trains and it schedule

dmgI received some question for my exam preparation, but I'm not sure about the answer. So I need some reasonable explanation of the following tasks: I. Every day Sonja arrives at the railway station between 6 and 7 pm, but her exact time of arrival is random within this one hour interval. The trai...

Words such as question do not add information to titles. Please edit the title so that it better describes the specifics of your question. Do not hesitate to make it longer or include a formula if needed. More tips here. (autocomment)Normal Human 21 secs ago
0
Q: Conformal Projections in ball

DiegoI need an exemple of the conformal aplication of $n$-dimensional ball $B^n \subset R^n$ in ball $n-1$ - dimensional $B^{n-1} = \{ (x_1,...,x_n) \in R^n ; x_1^2 + ... + x_{n-1}^2 = 1 and x_n = 0 \}$. Can some help me pleaser?

0
Q: L equivalence strings

VaroagLet L be the language consisting of all strings in (a+b)* that have an even number of letters and do not have aaba as a substring. Into how many L-equivalence classes is (a+b)* divided?

0
Q: Testing series for convergence Help

courageuxI have the following series, Series I have tried comparing with the limit comparison test to a p series. But the limit is hard to evaluate. Any suggestions to prove convergence? Thanks

Welcome to Math.SE, courageux. Words such as help are uninformative in titles. Please edit the title so that it better describes the specifics of your question. Do not hesitate to make it longer or include a formula if needed. More tips here. (autocomment)Normal Human 21 secs ago
0
Q: How to take definite integral of unspecified function in Mathematica

manofbearFor example, $\int_a^b (c-x)^2 g(x)\,dx$, where $g(x)$ is left unspecified. I have a bunch of terms like this and I want to see if it all adds up to $0$.

Questions tend to get more attention when they have a tag for a broad area of mathematics relevant to the question. Some of these tags might fit. (from a bot)Normal Human 21 secs ago
0
Q: problem about periodicity of function

Muhsin Ibn Al AzeezThis problem arised while I'm reading [this]( physics.stackex change.com/questions/221972/ combination-of-simple- harmonic-motions ) question on physics stackexchange. But it seems this platform is good for asking my doubt. In the above mentioned question, author asked about the periodicity of fu...

0
Q: Dyadic Green's Function

Soumyajit RoyWhat is dyadic Green's function? Is it related to coupled ODEs? Please refer to some good texts.

Short title. Short question. Question contains please. Dyadic Green's Function
0
Q: $R_1=\{(x,y) \in R^2:-1 \le x \le 1,-3 \le y \le 2 \}$

LegolasWe have the following relation: $R_1=\{(x,y) \in R^2:-1 \le x \le 1,-3 \le y \le 2 \}$ Could anyone tell me how to make the graph for the above relation?

A title should not be all-MathJax; having some plain text helps with search and navigation. (from a bot)Normal Human 21 secs ago
0
Q: $H_0(\Delta^m, \partial \Delta^m)$

user72870Let $\Delta^n$ be the standard $n$-simplex. Denote with $H_0$ the (simplicial) $0$-homology. In my book it is written that $H_0(\Delta^n, \partial \Delta^n)=\mathbb{Z}$. But $\Delta^n$ and $\partial \Delta^n$ have the same number of vertices, so $H_0(\Delta^n, \partial \Delta^n)=0$. Is it just ...

A title should not be all-MathJax; having some plain text helps with search and navigation. (autocomment)Normal Human 21 secs ago
 
6:12 PM
2
Q: Ability to close questions as dupe even if there are no answers

Tim BJust because a question has no answer doesn't mean it isn't a duplicate, so why can't we close them like that? For example: How to know if a page loaded via iframe is within sandbox? Detect if JavaScript is Executing In a Sandboxed Iframe? These are clear duplicates but we can't close either ...

 
0
Q: What's the point with the Gelfand–Naimark theorem?

Dac0Does anybody can explain me in plain english what's the real point with the Gelfand–Naimark Theorem. I know it's crucial, but I think I'm missing how much it's crucial.

0
Q: Inequality of a finite sum

TilingIm reading a book about and discoverd a inequality, i have Problems with. $$(\sum_{2^l\leq|m|<2^{l+1}}|\hat{f}(m)|)^2\leq(\sum_{2^l\leq|m|<2^{l+1}}1^2)\sum_{2^l\leq|m|<2^{l+1}}|\hat{f}(m)|^2$$ Surly it is true that: $$(\sum_{2^l\leq|m|<2^{l+1}}|\hat{f}(m)|)^2\leq\underbrace{(\sum_{2^l\leq|m|<

0
Q: Prove if the sum converges

F.leonDon't know how to prove if converges or not. which converging test do i need to use? $\sum_{n=1}^{\infty} \frac{(1}{nln^2(n)}

Short title. Short question. Prove if the sum converges
0
Q: Solve system of n equations

bobby42Solve the system of n equations: $$ \begin{cases} 2x_1^3+4=x_1^2(x_2+3)\\ 2x_2^3+4=x_2^2(x_3+3)\\ .........\\ 2x_{n-1}^3+4=x_{n-1}^2(x_n+3)\\ 2x_n^3+4=x_n^2(x_1+3)\\ \end{cases} $$

Short title. Short question. Solve system of n equations
 
6:29 PM
0
Q: Poncelet's Porism

aGer Let $E_0$ and $E_1$ be two ellipses contained in the projective plane $\textbf{P}$. Each ellipse bound a disk on one side and a Möbius band on the other. We assume that the disk bounded by $E_1$ contains $E_0$. We normalize by a projective transformation so that $E_0$ is the unit circle and $E...

Short title. Poncelet's Porism
0
Q: Integration of Nonnegative funtion, Real Analysis Chapter 2 problem 16

Morgan WeissIf $f\in L^{+}$ and $\int f < \infty$, for every $\epsilon > 0$ there exists $E\in M$ such that $\mu(E) < \infty$ and $\int_{E} > \left(\int f\right) - \epsilon$ Proof: Let $\epsilon > 0$, and $f\in L^{+}$ where $f$ is simple, we can define $$\int f d\mu = \sup\{\int \phi d\mu: 0\leq \phi \leq f...

Title contains problem. Tagged proof-verification. Integration of Nonnegative funtion, Real Analysis Chapter 2 problem 16
0
Q: Convert a surface to cylindrical coordinates.

TomI have 9z=−2x+7y and I need to convert it to cylindrical coordinates in the form of r=f(θ,z)=____ I started by substituting x=rcos(theta) and y=rsin(theta) in but got confused as to what to do next.

This site uses MathJax formatting of formulas. More tips here. (autocomment)Normal Human 21 secs ago
0
Q: Solution check: How to find the number of elements of order p and sylow p-subgroups in Sp?

zagadka314This is my original problem: Find the number of elements of order 7 in $S_7$. Find the number of Sylow 7-subgroups of $S_7$. Since 7 is prime, we know that this must be a 7-cycle in $S_7$ (since no $P_1 * P_2 = 7$). My next thought is to calculate the total number of combinations a number ...

0
Q: Computing $\lim_{n \to \infty} \sup_{x \in (0,1)}\left| \frac{1}{nx} \right|$

WillI believe that the above is equal to $\infty$ but I don't have any formal reason why.

0
Q: Quotient groups of D10

saraHow do I find the groups D10/N where N is the normal subgroups to D10? I know that the definition is aN for all a $\in$ D10. But I am unsure what the group, for example, D10/ D10 looks like? Any help would be much appreciated.

0
Q: Graph Theory problems

Eric SchaalI have an assignment for next week and I'm stuck with these two questions : a) Let G be a simple graph on 8 vertices with exactly 25 edges. Can G be Eulerian? How about with 24 edges? What I did : a) A graph on 8 vertices contains at most C(8,2)=28 edges. So G is a complete graph -3 edges. The...

Short title. Title contains problem. Graph Theory problems
0
Q: Let R be the relation on J6 defined by R = { (a, b) ∈ J6 × J6 | a = b or a + 3 ≤ b }. Prove that R is a partial order relation on J6.

FranSo I fully understand that a=b is a partial order relation as it is the identity relation. a+3<=b however is where I am being stumped.

0
Q: Hilbert Spaces and Banach Spaces

Javier Rojas MadrigalI have a problem with the definition of Hilbert Space and Banach Space. What is the difference between a Hilbert Space and a Banach Space?

0
Q: Finite Subgroup of $(Q,+)$ is cyclic.

Rishabh SareenProve that any finitely generated subgroup of $(Q,+)$ is cyclic.

0
Q: For $f(x)$ on $(a,b)$, s.t. $f'(x)+f(x) \ge -1$ and $\lim\limits_{x \to a} f(a) = - \lim\limits_{x \to b} f(a) = \infty$, then $b-a \ge \pi$

MonstrousMoonshinef(x) is continuously differentiable here. Using separation of variables, I think I might have shown that the equality form of the statement is true, but I'm a bit wary of trying separation of variables for a differential inequality. There was no solution provided for the problem.

Consider replacing (analysis) with a more specific tag for the relevant branch of analysis. (autocomment)Normal Human 21 secs ago
0
Q: Why does $b^{log_bx} = x$?

Ghost_StarkWhy does $b^{log_bx} = x$? Can someone break this down by showing me the steps as to why this is true?

Short title. Short question. Why does $b^{log_bx} = x$?
0
Q: Coordinate-free definition of elementary divisors

Jakob WernerThere is a general mantra in math which says that what is independent of bases shall be defined independent of bases. Well, it is well known that the elementary divisors of a linear map $M\xrightarrow{\ \ f\ \ }N$ of finitely generated free modules over a principal ideal domain $R$ are independen...

 
7:10 PM
1
Q: New nav seems to forget items per page

Jim GarrisonFor years I had a link on my browser toolbar that brought up a filtered list of questions. Once I selected 50/page I never had to reset the items/page unless I logged out (which I never do deliberately). With the new nav that link has become http://stackoverflow.com/questions/tagged/java%20xml...

 
0
Q: Why there is no fixed point for $f:S^n\to S^n$ when $n\ge 1$

grayQuantI can find a continuous map that has no fixed points for the case $n=1$ but fail to see how this generalizes.

0
Q: Represent one string of infinite length

escapGiven an alphabet $\Sigma = \{0, 1 \}$, to represent the set of infinite (bit) strings, it is usually used the notation $\{0,1\}^{\omega}$. To represent the fact that I take an element of this set, it is of course used the notation $str \in \{0, 1 \}^{\omega}$. What I need is to make explicit t...

Welcome to Math.SE, escap. Tag (notation) should not be the only tag a question has. Please add a tag for a subject area to which the question belongs. (autocomment)Normal Human 21 secs ago
0
Q: Equilibrium Points of the SIR Model

Ewan AndersonWhen considering the SIR model, are there any equilibrium points other than when the whole population is recovered?

 
0
Q: Why was my (topology) question closed

grayQuantShow that a matrix has strictly positive real eigenvalues topology exercise I don't understand, is the question too trivial? It's always very hard to know whether a question I ask will be well received or not.

 
7:28 PM
0
Q: Combining deterministic and stochastic asymptotics

STFConsider a sequence of real-valued random variables, $\{X_n\}_{n}$. Suppose that $X_n \rightarrow_p X$. Consider (*) $\lim_{n \rightarrow \infty} P(X_n>0)$. What I have seen doing in several books is the following: since $X_n\rightarrow_p X$ then (**) $\lim_{n \rightarrow \infty} P(X_n>0...

 
0
Q: stacksnippets error?

guest271314Below message intermittenly logged to console at SO , yesterday and today Error while parsing the 'sandbox' attribute: 'allow-modals' is an invalid sandbox flag. jquery.min.js:4f.extend.clone jquery.min.js:4e.fn.e.init jquery.min.js:2e jquery.min.js:2i.generate snippet-javascript.en.js?v=efa2f...

0
Q: Should I avoid using bold letters in question body?

StarkeenI have seen many old and new posts on stackoverflow where posters use/used bold texts to discribe their problems or to greet other users " Hello everyone! " I have an old version of apache server. mod_rewrite is disabled. How can I rewrite A to B? I tried ..... but it returned 500 error. What ...

 
0
Q: Convergence rate of the sequence $a_{n+1} = a_n-a_n^2, a_0=1/2$.

yoyoyoeThe sequence converges to zero at a rate that seems to be slightly faster than $1/n$. What are the best known results on the convergence rate of this sequence?

0
Q: What is e^(it) or e^(-it) equal to?

Enea What is e^(it) or e^(-it) equal to? In the example he finds the eigenvalues r=i and r=-i and for the general solution he writes: y=c1cost+c2sint.

0
Q: Riemannian manifolds isometry

user223794Here is the following problem: Let $g_0$ be the Euclidean metric on $\mathbb C=\mathbb R^2$. Let $M=\{z \in \mathbb C| \ |z|<1 \}$ and equip it with the Riemannian metric $g=\frac{1}{(1-|z|^2)^2}g_0.$ Let $N=\{z \in \mathbb C| \ \text{Im} \ z>0 \}$ and equip it with the Riemannian metric $...

0
Q: Prove that $G$ is Hamiltonian

Eric SchaalI'm stuck with this question. 1. Let G be a simple graph on 21 vertices with at least 200 edges. Show that G is Hamiltonian. I tried to use Dirac's theorem to prove it but it is inconclusive because I get δ(G) = 10 < 21/2. And I don't think we have seen Ore's theorem in class. Is there another...

0
Q: Can you always decrease the variance by removing outliers?

dash2Consider a finite set of real numbers. Let its variance be $V$. Let the highest number be $h$ and the lowest number be $l$. Let $x$ be an arbitrary number with $l < x < h$. Now create a new set by removing one element equal to $h$ and replacing it with $x$. Call the variance of this new set $V...

Welcome to Math.SE, dash2. Consider adding a tag for a broader subject area to which the question belongs. Some of these tags might fit. (from a bot)Normal Human 21 secs ago
0
Q: Group $a^6=b^{11}.$

Rishabh SareenLet $a$ and $b$ belong to a group $G$. If $o(a)=12$, $o(b)=22$ and $<a> \cap <b> \neq \{e\}.$ Prove that $a^6=b^{11}.$

Short title. Short question. Group $a^6=b^{11}.$
0
Q: Joint Density - statistics

JasonX and Y are independent and identically distributed exponential random variables with parameter 10. Compute the joint density of (U,V) where U=X+Y and V=exp(X).

Short title. Short question. Joint Density - statistics
 
8:07 PM
0
Q: Conditions on limit

VrouvrouPlease I have this two conditions: $$(H_1)~~\lim_{|t|\rightarrow0}\frac{f(t)}{g_1(t)|t|}=0,\,\,\,\, (H_2)~~\lim_{|t|\rightarrow\infty}\frac{f(t)}{g_2(t)|t|}=0$$ $$(H_3)~~\lim_{|t|\rightarrow0}\frac{B(t)}{G_1(t)}=0,\,\,\,\, (H_4)~~\lim_{|t|\rightarrow\infty}\frac{B(t)}{G_2(t)}=0$$ such that $$G...

Short title. Conditions on limit
0
Q: Constructing a Minimal DFA from L = (ab + b)* ba using Brzozowski's derivatives method

k-RockerHow would I use Brzozowski's derivatives method to construct a minimal DFA recognizing the language defined by the rational expression: L = (ab + b)* ba

0
Q: Choosing dices out of a box

mkropkowskiIn a box we have 5 dices, 3 of them are proper, one has two sixes, second has six sixes. Given the events A - after first throw we have a six, B- after next two throws we have six and something else. Find probability: $P(A \cap B | proper dice)$. My attempt is: $$ (1/6)\times(1/6)\times(5/6)=(5/6...

0
Q: Cycle colour without doubling

Arthen KuitrlWe have simple cycle graph $G$ of $9$ vertices. I colour $3$ vertices blue, $3$ red, $3$ green. We assume the same graph coloring $G'$ and $G''$ if $G'$ = $R \circ G''$ , $R$ - rotation. Let coloring is right if every two adjacent vertices have different colour. enter image description here ...

0
Q: Basis in Linear Transformations

CaydeI'm pretty dim when it comes to basis and span, please take that into account :) Help me prove this theorem: Let $V$, $W$ be vector spaces, also $T:V \rightarrow W$ is a linear transformation. If $\beta = \{v_1, v_2, ..., v_n\}$ is a basis for $V$ then $R(T) = span(\{T(\beta)\}) = span (\{T(v...

Question contains please. Basis in Linear Transformations
0
Q: A proof that BS(1,2) is not polycyclic

RonaldI am looking for examples of finitely generated solvable group that is not polycyclic. In Wikipedia Baumslag-Solitar group $BS(1,2)$ is an example. But how to prove this fact?

0
Q: Help me understand a proof for $=e^{A+B}=e^{A}e^{B}$

bluemoonI want to prove that $=e^{A+B}=e^{A}e^{B}$ using differential equations. I found a proof here: LINK Here is how the proof goes: Given a square matrix $M$, the function $X(t):=e^{tM}$ is the unique solution of the linear differential equation: $X'=MX$ and $X(0)=I$. Now set $X(t):=e^{tA...

Words such as help do not add information to titles. Please edit the title so that it better describes the specifics of your question. Do not hesitate to make it longer or include a formula if needed. More tips here. (from a bot)Normal Human 20 secs ago
 
8:34 PM
0
Q: E-Mail Spam claims I got them interested on a SE site

naltiparI got an email stating I got the OP interested through webapps.stackexchange.com. The e-mail reads: Nice meet you, how you doing? I hope am not offending you by message you got me interested here on webapps.stackexchange.com; i we love to have a good communication with you i we send you my p...

 
0
Q: Automobile license numberplate digit sum

NarasimhamFour digit vehicle numbers sum up to 18 ...due to a maximum probability from central value as: $$ (0+9)/2 *4 $$ Is it correct?

0
Q: Canonical class of blow-up

se0808 Let $B_1 \to X_1=\mathbb P^n$ be the blow-up along a linear subspace $\mathbb P^k$. Let $B_2 \to X_2$ be the blow up of a quadratic cone $X_2$ in its vertex. I need to calculate $K_{B_i}$. As any blow up is an isomophism ouside of exceptional locus, $K_{B_i}=\pi^*K_{X_i}+D_i$, where a divisor ...

0
Q: How to analyze CDF plot?

Naga Let $z$ be a random variable, which is defined as $ Z = d^{\alpha}$. where $d$ is random variable. I tried to plot the CDF for $Z$ with different values of $\alpha$, which is: So, average value(50th percentile) of $Z$ decreases with increase in $\alpha$ values. Is it correct? Because if we in...

0
Q: calculate $\lim\limits_{(x,y) \to(0,0)} \frac{x^4y^2}{ (x^4+y)^5}$

royI need to calculate $\lim\limits_{(x,y) \to(0,0)} \frac{x^4y^2}{ (x^4+y)^5}$ I get $[0/0]$. i think it doesn't have a limit but i don't know how to prove it. Thank you.

0
Q: Simplest Proof of the Dimensions theorem

CaydeThe theorem goes: Let $V$ and $W$ be vector spaces and $T:V \rightarrow W$ is a linear transformation. If $V$ is finite dimensional, then $nullity(T)+rank(T)=dim(V)$

0
Q: Task to check normal distribution and analyse confidence intervals

RLearnsMathI've got another question about normal distribution and its confidence intervals interpretation. Your explanation will respectively help me to better prepare for my examination. One of the social surveys, which selects a nationally representative sample of the Canadian adult population, asked re...

 
8:56 PM
0
Q: Definition of Paradoxical set

Kudera SebastianHi I would like to understand Banach-Tarski paradox, but well... my knowledge with set theory is very very limited. I know what a union is, I will tell you how to partition a simple set, know couple of things about sets being disjoint and well, that will be it. ANd every idea of proof (take awa...

0
Q: About a solution in Wolfram Alpha

AtaulfoI'm intrigued by this finding on the extraordinary portal WolphramAlpha. What is the reason why the solution has not been simplified, eliminating the factor $\frac{\sqrt x \sqrt{x+2}}{\sqrt {x(x+2)}} $? If not removal is justified, I would love to know why (I not discard that there is a strong...

Tag (soft-question) should not be the only tag a question has. Please add a tag for a subject area to which the question belongs. (autocomment)Normal Human 20 secs ago
0
Q: Finding a potential

Cure $F(x,y)= \left(\displaystyle\frac{1-y^2}{(1+xy)^2},\displaystyle\frac{1-x^2}{(1+xy)^2}\right)$. I've been having some troubles to find the potential of $F$. To find it the idea was finding $\int F dx = g(x,y) +h(y)$, and having $g$ find the function $h$. Unless my calculations have a mistak...

Short title. Tagged differential-equations but mentions "partial". Finding a potential
0
Q: Graph theory - the count of edges

user288083How much the most edges have graph on $n$ vertices with 2 components? How much the most edges have graph on $n$ vertices with 3 components? Thank you for any hints.

0
Q: Algorithm to construct matrix

VWasquesI am trying to do an algorithm to construct the follow matrix $\begin{bmatrix} 0 & 1 & 2 & 3 & 0 & 1 & 2 & 3 & 0 & 1 & 2 & 3 & 0 & 1 & 2 & 3 \\ 0 & 0 & 0 & 0 & 1 & 1 & 1 & 1 & 2 & 2 & 2 & 2 & 3 & 3 & 3 & 3 \end{bmatrix}$ This is what I did: First I construct the first line of the matrix: $M(1...

0
Q: Discreet Problem with Equiv Classes

Eugene WalesI'm not even sure where to start. The problem is: ====================================================== Consider the subset H = { [3k] | k ∈ Z } of Z12. (a) Determine the distinct elements of H and construct an addition table for H. (b) A relation R on Z12 is defined by [a] R [b] if [a-b] ∈...

Title contains problem. Tagged proof-writing. Discreet Problem with Equiv Classes
0
Q: Help to solve $\nabla \cdot(\nabla F(x))$

user295656I'd like to find $\nabla \cdot(\nabla f(x))$ where $r=\sqrt{x^2+y^+z^2}$. I know that $\nabla f(x)=\partial_xf\vec{e_x}+\partial_yf\vec{e_y}+\partial_zf\vec{e_z}$. I don't underastnt how use this for my case when it's know what $r$ is equaled.

Welcome to Math.SE, user295656. Words such as help are uninformative in titles. Please edit the title so that it better describes the specifics of your question. Do not hesitate to make it longer or include a formula if needed. More tips here. (from a bot)Normal Human 21 secs ago
0
Q: quintic diophantine equation

user84909Find a (parametric) infinite family of solutions for the equation $a^5+b^5+c^5+d^5+e^5+f^5=a+b+c+d+e+f$, where $a,b,c,d,e,f$ are integers. Numerical experimentation suggests that this should be possible. For example, there are known parametric solutions for the equation $a^5+b^5+c^5+d^5+e^5+f^...

0
Q: Calculatin $\mathbb{E}X^2$

mkropkowskiWhen we could use the following equation: $$\mathbb{E}X^2=\int_0^\infty 2t \mathbb{P}(X>t)$$ I mean how is it possible to change $X^2$ to $2t$?

Short title. Short question. Calculatin $\mathbb{E}X^2$
0
Q: What Are Different Approaches to Introduce the Elementary Functions Rigorously?

H. R.Motivation We all get familiar with elementary functions in high-school or college. However, as the system of learning is not that much integrated we have learned them in different ways and the connections between these ways are not clarified mostly by teachers. Once I read the calculus book by ...

0
Q: If $i^2=-1$, at one point undefined, then why not define $\frac00$?

Jacob WheelerWould this even assist math in the way that $i$ did? Or is this just outright pointless and/or too exclusive to call for a definition?

Consider replacing (analysis) with a more specific tag for the relevant branch of analysis. (autocomment)Normal Human 21 secs ago
0
Q: Convergence of series (square root and nth power)

AleIs the series $$\sum_{n=1}^\infty\left(1-\frac1{\sqrt n}\right)^n$$ convergent?

0
Q: Variables and exponents

G SkeetHow would you solve this equation? 500n = 4000(1.016)^n I tried using some logarithms but I could not do it. The only unknown variable is n but I'm having a bit of trouble getting there. Thanks.

Short title. Random balls in urn
0
Q: Functional equation g(n)=f(f(n))+1

John J.Two functions f:N→N and g:N→N meet the following conditions. (1) For A={f(n)|n∈N}, B={g(n)|n∈N}, A∩B=Φ (2) A∪B=N (3) g(n)=f(f(n))+1 for n∈N (4) f(n+1)>f(n), g(n+1)>g(n) Question: Find f(240) This is a math contest problem but I have no idea. I tried: f(1)=1 or g(1)=1 i) If f(1)=1: g(1)=f...

This site uses MathJax formatting of formulas. More tips here. (autocomment)Normal Human 21 secs ago
0
Q: Random balls in urn

Albert1) Two boxes, both contain $6$ white $\&$ $5$ black balls. Random ball $A'$ is drawn from the first box and place into second box. Then random ball $A''$ is drawn from the second box. What is the probability that $A$ is white? 2) Three boxes, the first contains $3$ white $\&$ $4$ black balls,...

0
Q: Solution for Sine Equation

Evokecould you please help me with this equation? 3sin(3x+π/4)=0 I know that the answer is obviously π/4 (or k*2π + π/4). but could you give me a step-by-step solution? Could you also tell me how this equation is solved? sinh(x+1)=1 Thank you very much.

Welcome to Math.SE, Evoke. This site uses MathJax formatting of formulas. More tips here. (from a bot)Normal Human 21 secs ago
 
9:44 PM
0
Q: Why is the dimension of $k$-Tensors that are antisymmetric $\binom{n}{k}$

AmontilladoI'm trying to prove it but so far I can only write the antisymmetric tensor using the basis for all $k$-tensors. Can you please help me?

0
Q: Building agglomerative hierarchical clustering algorithm manually

User199932I am currently trying to solve this problem but I'm quite unsure if my answer is correct. I've build the tree different types of measures, MIN, MAX and AVERAGE but I might have some of them wrong. Can anyone check and tell me for sure please. Thanks in advance. Image 1 contains the maths problem...

0
Q: Improper Integral comparison type question

Seojun HongSuppose $(a_n)$ is a sequence in $\mathbb{R}$ such that $a_n > 0$ for all $n \in \mathbb{N}$ and that there exists $f: [0, \infty) \to \mathbb{R}$ such that $f(n) = a_n$ for all $n \in \mathbb{N}$. Assume that $f$ is a continuous function and decreasing. Also $f > 0$. I would like to prove the ...

Words such as question do not add information to titles. Please edit the title so that it better describes the specifics of your question. Do not hesitate to make it longer or include a formula if needed. More tips here. (from a bot)Normal Human 21 secs ago
0
Q: Uniformly Continuous Sequences

user282934How can I show: Let {$f_n$} be a sequence of functions that are uniformly continuous on $(0,1)$. Show if {$f_n$} converges uniformly on $(0,1)$ then $f(x)$ is uniformly continuous on $(0,1)$. Also, would someone mind explaining what it means to have a uniformly continuous sequence?

0
Q: Filter two matrices in Matlab

AMIRI have two matrices $A_{a\times n}$ and $B_{b\times n}$ (as it is apparent from their sizes, they have the same number of columns). Now I want to filter each column of $A$ by its corresponding column in $B$ in Matlab without using for-loop (in a vectorized manner). Indeed I want to filter the fir...

Short title. Tagged matlab. Filter two matrices in Matlab
0
Q: Can someone please explain this concept (Improper Integrals)

Joffrey BaratheonSo according to my math textbook the integral of ln|x+1/x+2| from 0 to Infinity is equal to ln(2). I don't understand how the limit of ln|x+1/x+2| as x approaches infinity is zero. I cannot find any explanation in the textbook.

Welcome to Math.SE, Joffrey Baratheon. This site uses MathJax formatting of formulas. More tips here. (autocomment)Normal Human 21 secs ago
 
10:13 PM
0
Q: Prove that there exists an isometry P in Isom(H^2) such that P(A)=B

John.PWhere A and B are ideal triangles in H^2 (upper sheet of hyperboloid). How do I get started with this proof?

0
Q: Significance of the roots of the zeta-function

Tommy Räjerti've been looking for some applications or some kind of usage for the zeta-function an what a proof of the Riemann hypothesis would mean to areas such as number theory, but I haven't been successfull researching the latter. Could someone go through some important things relying on the proof of th...

Questions tend to get more attention when they have a tag for a broad area of mathematics relevant to the question. Some of these tags might fit. (autocomment)Normal Human 21 secs ago
0
Q: Testing series for divergence

courageuxI have the following series: Sum from one to infinity ln(2(n+1))- ln(2n) How can I test it to show it is divergent?

Short title. Short question. Testing series for divergence
0
Q: How many 7 letter words can be formed from the letters in bookkeeper?

NaokiI was thinking that i would write it out like this (10/3)(8/2)(6/2)*5 factorial. but i realized that would just show the total possible ways the word can be rearranged. if i was to guess as to what to do i would say that (10/7). which translates to (10factorial)/7factorial *3factorial)

This site uses MathJax formatting of formulas. More tips here. (autocomment)Normal Human 21 secs ago
0
Q: What is the largest abelian subgroup in S_n?

Geoffrey CritzerI am almost to complete my first course in group theory. I have read Dummit and Foote into chapter 5. I know that I can always find an abelian subgroup isomorphic to C_k(1) X C_k(2) X ... X C_k(j) in S_n where n = k(1) + k(2) + ... + k(n). Specifically, the permutation group G = <(1 2...k(1)),...

This site uses MathJax formatting of formulas. More tips here. (autocomment)Normal Human 21 secs ago
0
Q: Nilpotency class of subgroup

user114539Let $G$ be a group of nilpotency class $n$. Every proper subgroup of $G$ would have nilpotency class at most $n$. Do we know of an example where the commutator subgroup of a nilpotent group of class $n$ also has class $n$?

 
10:39 PM
0
Q: POLL: Should we participate in the 2015 “Winter Bash” Holiday hats promotion?

StrongBadIn 2015, Stack Exchange will continue its tradition of the "Winter Bash". Winter Bash is an annual event that can run on any Stack Exchange site that chooses to participate. Users earn “hats” for their gravatars by completing certain tasks (analogous to badges). Certain actions trigger the user r...

0
Q: Question tag change after answering it

AshiquzzamanThis question is originally tagged with java. So I answered the question based on java. But later OP edit tag and tag it with c. What is your suggestion about handle this situation?

 
0
Q: Finite Element Method for the 1d wave equation

Fryderyk WilczynskiI'm solving the 1D wave equation \begin{equation} \frac{\partial^2 \eta}{\partial t ^2} - \frac{\partial^2 \eta}{\partial x ^2} = 0 \end{equation} with boundary conditions \begin{equation} \frac{\partial \eta}{\partial x} = 0 \qquad \qquad \text{on} \qquad \qquad x = 0, L \end{equation} using Fi...

0
Q: The proof of Newton's method quadratic convergence (Taylor's theorem)

gigiFirst of all, why do we take the absolute value of both sides? What is the point/reason? and for (x_n - z)^2 , isnt it always positive? Second of all, do you call this a recurrence relation? for the last part: e[n+1]=Ce2ne[n+1]=Cen2 but how can we conclude that the relations is telling us the ac...

This site uses MathJax formatting of formulas. More tips here. (autocomment)Normal Human 21 secs ago
0
Q: Given the function f(x,y) = e^(-y^2)*(2x^3-3x^2+1) + e^(-y)*(2x^3-3x^2), how do I show that the function has no global maximum?

nothingtodohereI have already found that (0,0,1) is a local maximum (and the only critical point), but I cannot figure out how to show that there is no global max.

0
Q: Random partition

AlbertLet $A$ be set of $n = 9k$ elements. Let $(A_1,A_2,A_3)$ be partition of $A$: $A = \cup_i A_i$, $|A_i|=\frac{n}{3}=3k$, $|A_i\cap A_j| = 0$, $i\neq j$ We assume two partitions same if set collections for partitions are same. For example $(A_1,A_2,A_3)=(A_2,A_1,A_3)$. Let $B$ be subset of $...

Short title. Random partition
 
0
Q: Suggested edits notifcation not taking filter into account

RobCurrently, the pending suggested edits icon next to review does not take into account your filter for that review queue. This results in having the review icon permanently showing, and hides the instances of pending suggested edits I'm actually interested in. Could we have this notification sh...

 
0
Q: Simple definite integral

TheBlueWizardI do not figure out how to solve $$ L= \frac{1}{2}\int_1^4 \sqrt{(16t^2+t+4)}\;dt $$ The key is probably in simplifying the polynomial equation, but I don't find a way that is really simplifying the integral. Can someone help? Thank you.

0
Q: Proving That a Parallelogram is a Rectangle

halcyon_kGiven a parallelogram with congruent diagonals, you are asked to prove that the parallelogram is a rectangle. Would saying: A parallelogram must be a rectangle if the diagonals are congruent. be a valid proof for this question?

Welcome to Math.SE, halcyon_k. Tag (proof-verification) should not be the only tag a question has. Please add a tag for a subject area to which the question belongs. (autocomment)Normal Human 21 secs ago
0
Q: Use of the "Optimality Principle" in a problem regarding maximization of a product indexed by a partition of a natural number.

smnels11I am currently reading a paper (Iterated Binomial Coefficients by S.W. Golomb, The American Mathematical Monthly, 1980, 719-727) that makes use of the "optimality principle" in a couple of proofs. One theorem in which they rely on it is as follows: Let $n$ be a positive integer and define $g(n) ...

 
10:57 PM
1
Q: Are visa and study permits considered off topic?

The HiaryWould it be considered off topic if I ask questions about F1, J1 visas and study permits? From my experience the travel stackexchange community frowns upon such questions.

 
0
Q: Effective Methods of Studying in different areas of Math

RedI apologize if this question isn't appropriate for this site but I am looking for advice I think other math students might better be able to give me. I am an undergraduate math major about enter into my second year. What I have found is that I love the theoretical discussions of mathematics an...

0
Q: Sum set probability

IvanLet build a set A by this rule: for $i$ from $0$ to $n - 1$ with probability of $\frac{1}{2}$ we add this number to set $A$. Then we generate set $B = A + A := \{a_1 + a_2: a \in A, a_2 \in A\}$ And the question is to find upper boun on probability $P(k \notin B)$ as precise as possible. I h...

Short title. Sum set probability
0
Q: Isomorphism and $(Q,+)$

TheNotMe Prove that $(Q,+)$ is not isomorphic to $(H,+) \neq (Q,+)$, a subgroup of $(Q,+)$. $Q$ is the rationals. I thought about taking the contradiction direction. If we do that then we have $f:Q\to H$ such that $f$ is an isomorphism. That means that that it is surjective and injective, but $(H,+)...

0
Q: Equation with different bases (exponential)

FermiumI seem to be stuck with this one right here: 2^x + 2^x+1 = 3^x+2 + 3^x+3 Mind to help me with it?

Welcome to Math.SE, Fermium. This site uses MathJax formatting of formulas. More tips here. (autocomment)Normal Human 20 secs ago
 
11:13 PM
-3
Q: 5,000 rep. Where can I approve/reject tag wikis?

bytecode77So I have 5,000 rep now and I'm seemingly something special now... But I still didn't find the place where I can use my super powers. I've seen this page, but there's still no clarity for me. Where do I approve or reject tag wiki edits?

2
Q: Is this really a good audit question?

MarvinI just failed this first post audit by downvoting it. It seems that the exception was good enough for others to solve the issue and I saw other meta questions indicating that a short question is perfectly fine if it contains enough information, so I wasn't sure if it should be closed. And while...

1
Q: How are we protecting our heads this year?

Daniel FischerOnce again, the time of the year has come when we have to decide whether we want to participate in the Winter Bash. The event will start on 14 December 2015 and run up to and including 03 January 2016. Users will be able to see all the hats they've earned on http://winterbash2015.stackexchange.c...

 
0
Q: Proof with Equiv Relation and Equiv Classes

Eugene WalesConsider the subset H = { [3k] | k ∈ Z } of Z12. (a) Determine the distinct elements of H and construct an addition table for H. (b) A relation R on Z12 is defined by [a] R [b] if [a−b] ∈ H. Show that R is an equivalence relation and determine the distinct equivalence classes. No idea where t...

Short question. Tagged proof-writing. Proof with Equiv Relation and Equiv Classes
0
Q: Physics Help!!!!!!

user295684) Dan travels from his home to the store and back. On the way to the store, he travels at a speed of 50 km/hr for the initial half distance and 100 km/hr for the latter half. On the way back, he travels 50 km/hr for the first half time and 100 km/hr for the latter half. The total distance from hi...

Welcome to Math.SE, user295684. Words such as help are uninformative in titles. Please edit the title so that it better describes the specifics of your question. Do not hesitate to make it longer or include a formula if needed. More tips here. (autocomment)Normal Human 20 secs ago
0
Q: Proof on modular congruence

RexProve that for n in the set of natural numbers, n is greater thean or equal to 2: For all a belonging to the set of natural numbers, For all b belonging to the set of natural numbers, a is modular congruent(subscript n) to b -->a^2 (subcript n)b^2. Sorry for writing in words, i am new to the si...

Short title. Tagged proof-writing. Proof on modular congruence
0
Q: Condition for Sums in a Set

Yash PatelLet $$ S = \{ a_1,a_2, \cdots , a_k \}$$ be comprised of divisors of $n \in \mathbb{N}, n>1$ and $n$ not prime. Suppose we select $p$ elements from the set $S$ (possibly more than once for each divisor), and the $p$ chosen elements have a total sum of $q$. Prove that it is always possible to sel...

0
Q: Testing infinite series for convergence

courageuxI have the following series Sum from 1 to infinity ne^(-n^2) How would one go about showing convergence? Which test can be used?

 
11:46 PM
0
Q: prove $(A \times B)^c = (A^c\times U)\cup(U\times B^c)$

Amigot stuck proving that $(A \times B)^c = (A^c\times U)\cup(U\times B^c)$. would appreciate your help, this is what I got so far:

0
Q: Inverse of an exponential function

RobertTzareI am having difficulties forming the inverse of this f(x) = 3*2^(3x+1)*5^(3x-1) What I have done so far: 3*2^3y*2^1*5^3x*5^-1 3*2*1/5*(5*2)^3y 6/5*10^3y .ln ln6/5*ln10*3y That is all.

0
Q: Integration of Nonnegative function, Folland Real Analysis using MCT

Morgan WeissSuppose $\{f_n\}_{n=1}^{\infty}\subset L^{+}$, $\lim_{n\rightarrow \infty} = f$ pointwise, and $\int f d\mu = \lim_{n\rightarrow \infty}\int f_n d\mu < \infty$. Then $\int_{E}f d\mu = \lim_{n\rightarrow \infty}\int_{E}f_n d\mu$ for $E\in M$ Proof: Let $\{f_n\}\subset L^{+}$ and $f_n\rightarrow f...

This site uses MathJax formatting of formulas. More tips here. (autocomment)Normal Human 20 secs ago
 
00:00 - 14:0014:00 - 00:00

« first day (31 days earlier)      last day (543 days later) »